How do I prove the transitivity of a set of implications?












4












$begingroup$


If I have a set of implications, how can I prove the transitivity? In other words: I know the transitivity law, but I need to show on paper for an assignment whether the argument is valid or not.



$$
begin{align}
P&to Q\
Q&to R \
therefore P&to R
end{align}
$$



I recall something to do with assuming P and/or Q, since $Pto Q$ will always be true if P is false anyhow, and the same with $Qto R$... but I don't know how to do it on paper.










share|cite|improve this question











$endgroup$

















    4












    $begingroup$


    If I have a set of implications, how can I prove the transitivity? In other words: I know the transitivity law, but I need to show on paper for an assignment whether the argument is valid or not.



    $$
    begin{align}
    P&to Q\
    Q&to R \
    therefore P&to R
    end{align}
    $$



    I recall something to do with assuming P and/or Q, since $Pto Q$ will always be true if P is false anyhow, and the same with $Qto R$... but I don't know how to do it on paper.










    share|cite|improve this question











    $endgroup$















      4












      4








      4





      $begingroup$


      If I have a set of implications, how can I prove the transitivity? In other words: I know the transitivity law, but I need to show on paper for an assignment whether the argument is valid or not.



      $$
      begin{align}
      P&to Q\
      Q&to R \
      therefore P&to R
      end{align}
      $$



      I recall something to do with assuming P and/or Q, since $Pto Q$ will always be true if P is false anyhow, and the same with $Qto R$... but I don't know how to do it on paper.










      share|cite|improve this question











      $endgroup$




      If I have a set of implications, how can I prove the transitivity? In other words: I know the transitivity law, but I need to show on paper for an assignment whether the argument is valid or not.



      $$
      begin{align}
      P&to Q\
      Q&to R \
      therefore P&to R
      end{align}
      $$



      I recall something to do with assuming P and/or Q, since $Pto Q$ will always be true if P is false anyhow, and the same with $Qto R$... but I don't know how to do it on paper.







      logic propositional-calculus






      share|cite|improve this question















      share|cite|improve this question













      share|cite|improve this question




      share|cite|improve this question








      edited Jan 19 '13 at 16:10









      Namaste

      1




      1










      asked Jan 17 '13 at 18:17









      agent154agent154

      3,752216599




      3,752216599






















          6 Answers
          6






          active

          oldest

          votes


















          12












          $begingroup$

          $$(1) Prightarrow Qtag{Hypothesis}$$
          $$(2) Qrightarrow Rtag{Hypothesis}$$
          $$quadquadunderline{(3); Pquad }tag{Assumption}$$
          $$quad quadquad |(4) Q tag{1 and 3: Modus Ponens}$$
          $$quadquadquad |(5) R tag{2 and 4: Modus Ponens}$$
          $$(6) P rightarrow R tag{3 - 5: if P, then R}$$



          $$therefore ((Prightarrow Q)land (Qrightarrow R))rightarrow (Pto R)$$


          (Note: step 6 is sometimes justified by "conditional introduction": together with what you are given or have established, if by assuming P, you can derive R, then you have shown $P rightarrow R$).





          Note: when I first learned propositional logic, once it was established (proven), we referred to the following "syllogism":
          $$Pto Q\
          underline{Qto R}\
          therefore Pto Rquad$$
          by citing it as the "Hypothetical Syllogism", for justification in future proofs.






          share|cite|improve this answer











          $endgroup$





















            4












            $begingroup$

            Since the logic you are using is not quantified, and there are only three variables, you can prove the argument valid with an eight row truth table covering all of the possible truth values of the three variables:



            P  Q  R    Premise:  P -> Q    Premise:  Q -> R:    Conclusion:  P -> R
            ------------------------------------------------------------------------
            0 0 0 1 1 1
            0 0 1 1 1 1
            0 1 0 1 0 1
            0 1 1 1 1 1
            1 0 0 0 1 0
            1 0 1 0 1 1
            1 1 0 1 0 0
            1 1 1 1 1 1


            Now an argument is valid if the conclusion is true whenever all the premises are true.
            This means that in all the rows where both premises have a 1, we check whether the conclusion has a 1. By golly, this is the case. Therefore the argument is valid!



            If we found a zero, that would be a counterexample which destroys the argument: a situation where the premises are all true, yet the conclusion is false.



            In my table, the P -> Q column and its siblings are simply derived from the truth table for the conditional. P -> Q is only false when P is true, and Q is false, and true for the other three possible values of the variables.






            share|cite|improve this answer









            $endgroup$





















              3












              $begingroup$

              I'm not sure about what notations you might use nor if you're using a natural deductive system, but this is the idea behind what you asked:



              Suppose $Pto Q\
              Qto R\$ are true.



              We want to prove that $Pto R$ is true.



              To do this suppose $P$ is true.



              Because $Pto Q$ is true it follows that $Q$ is true.
              Now because $Q$ is true, from $Qto R$ being true follows that $R$ is true.



              We assumed $P$ was true and we deduced that $R$ is also true, therefore $Pto R$ as we wanted.






              share|cite|improve this answer











              $endgroup$













              • $begingroup$
                Is this just a form of proof by cases? I guess it's one half of it anyhow - knowing that if we assume $neg P$, the implication will be true no matter what - it's only relevant if we show that it's true if P is true?
                $endgroup$
                – agent154
                Jan 17 '13 at 18:28










              • $begingroup$
                No, it's not a proof by cases. What you mentioned is one way to do it: you always have $neg P vee P$, so you do it by cases the way you mentioned. However if you wanna prove a conditional statement, i.e., a statement of the form $Ato B$, it suffices to assume $A$ is true and then somehow deduce that $B$ is true.
                $endgroup$
                – Git Gud
                Jan 17 '13 at 18:32












              • $begingroup$
                I just realized - I could convert the implications to clausal form using the rule $(neg Pvee Q)wedge (neg Qvee R)Rightarrow (neg Pvee R)$ and then use the resolution rule to resolve out $(neg Qvee Q)$ to get $neg Pvee R$... That works as well.
                $endgroup$
                – agent154
                Jan 17 '13 at 18:38












              • $begingroup$
                Yes, that works too.
                $endgroup$
                – Git Gud
                Jan 17 '13 at 18:39



















              1












              $begingroup$

              Although it is late, I wanted to give a simple and convincing demonstration:



              (1) P -> Q (Hypothesis)
              (2) Q -> R (Hypothesis)
              (3) ¬Q -> ¬P (Contraposition (1))
              (4) (Q -> R) ∧ (¬Q -> ¬P) (Union (2) and (3))
              (5) Q ∨ ¬Q (Law of excluded middle)
              (6) (Q -> R) ∧ (¬Q -> ¬P) ∧ ( Q ∨ ¬Q) (Union (4) and (5) )
              ____________________________
              (R ∨ ¬P) (Dilemma of (6), P -> R=def.(R ∨ ¬P) Definition of logical implication)





              share|cite|improve this answer









              $endgroup$





















                1












                $begingroup$

                Denote by $+$ the $or$ logic operator and by $cdot$ the $and$ logic operator.



                We have $(ARightarrow B)=bar A+B$ and $(BRightarrow C) =bar B+C$



                Assume $(bar A + B)=1=(bar B+C)$.



                Then
                $1=1cdot 1=(bar A + B)cdot (bar B+C) \
                = (bar Abar B + bar A C + Bbar B + BC)\
                = bar A + BC
                = (bar A + B) cdot (bar A+C)
                \
                =bar A + C
                $






                share|cite|improve this answer











                $endgroup$













                • $begingroup$
                  Ha! This is a great argument
                  $endgroup$
                  – Ben Kushigian
                  Dec 23 '18 at 17:17



















                0












                $begingroup$

                The answer for $6$ is indeed, true but if you had to show it with a truth table would be as follows



                $$begin{array}{cccccccc}P & Q & R & P!rightarrow! Q & Q!rightarrow! R & P!rightarrow! R & (P!rightarrow! Q)!wedge!(Q!rightarrow! R) & [(P!rightarrow! Q)!wedge!(Q!rightarrow! R)]!rightarrow! (P!rightarrow! R)\
                T & T & T & T & T & T & T & T\
                T & T & F & T & F & F & F & T\
                T & F & T & F & T & T & F & T\
                T & F & F & F & T & F & F & T\
                F & T & T & T & T & T & T & T\
                F & T & F & T & F & T & F & T\
                F & F & T & T & T & T & T & T\
                F & F & F & T & T & T & T & Tend{array}$$





                As you notice above the column $(P!rightarrow! Q)!wedge!(Q!rightarrow! R)$ is the same as the column of $(P!rightarrow! R).$ So with both the hypotheses $(P!rightarrow! R)$ and $(Q!rightarrow! R)$ we can conclude that $(P!rightarrow! R).$






                share|cite|improve this answer











                $endgroup$














                  Your Answer








                  StackExchange.ready(function() {
                  var channelOptions = {
                  tags: "".split(" "),
                  id: "69"
                  };
                  initTagRenderer("".split(" "), "".split(" "), channelOptions);

                  StackExchange.using("externalEditor", function() {
                  // Have to fire editor after snippets, if snippets enabled
                  if (StackExchange.settings.snippets.snippetsEnabled) {
                  StackExchange.using("snippets", function() {
                  createEditor();
                  });
                  }
                  else {
                  createEditor();
                  }
                  });

                  function createEditor() {
                  StackExchange.prepareEditor({
                  heartbeatType: 'answer',
                  autoActivateHeartbeat: false,
                  convertImagesToLinks: true,
                  noModals: true,
                  showLowRepImageUploadWarning: true,
                  reputationToPostImages: 10,
                  bindNavPrevention: true,
                  postfix: "",
                  imageUploader: {
                  brandingHtml: "Powered by u003ca class="icon-imgur-white" href="https://imgur.com/"u003eu003c/au003e",
                  contentPolicyHtml: "User contributions licensed under u003ca href="https://creativecommons.org/licenses/by-sa/3.0/"u003ecc by-sa 3.0 with attribution requiredu003c/au003e u003ca href="https://stackoverflow.com/legal/content-policy"u003e(content policy)u003c/au003e",
                  allowUrls: true
                  },
                  noCode: true, onDemand: true,
                  discardSelector: ".discard-answer"
                  ,immediatelyShowMarkdownHelp:true
                  });


                  }
                  });














                  draft saved

                  draft discarded


















                  StackExchange.ready(
                  function () {
                  StackExchange.openid.initPostLogin('.new-post-login', 'https%3a%2f%2fmath.stackexchange.com%2fquestions%2f280893%2fhow-do-i-prove-the-transitivity-of-a-set-of-implications%23new-answer', 'question_page');
                  }
                  );

                  Post as a guest















                  Required, but never shown

























                  6 Answers
                  6






                  active

                  oldest

                  votes








                  6 Answers
                  6






                  active

                  oldest

                  votes









                  active

                  oldest

                  votes






                  active

                  oldest

                  votes









                  12












                  $begingroup$

                  $$(1) Prightarrow Qtag{Hypothesis}$$
                  $$(2) Qrightarrow Rtag{Hypothesis}$$
                  $$quadquadunderline{(3); Pquad }tag{Assumption}$$
                  $$quad quadquad |(4) Q tag{1 and 3: Modus Ponens}$$
                  $$quadquadquad |(5) R tag{2 and 4: Modus Ponens}$$
                  $$(6) P rightarrow R tag{3 - 5: if P, then R}$$



                  $$therefore ((Prightarrow Q)land (Qrightarrow R))rightarrow (Pto R)$$


                  (Note: step 6 is sometimes justified by "conditional introduction": together with what you are given or have established, if by assuming P, you can derive R, then you have shown $P rightarrow R$).





                  Note: when I first learned propositional logic, once it was established (proven), we referred to the following "syllogism":
                  $$Pto Q\
                  underline{Qto R}\
                  therefore Pto Rquad$$
                  by citing it as the "Hypothetical Syllogism", for justification in future proofs.






                  share|cite|improve this answer











                  $endgroup$


















                    12












                    $begingroup$

                    $$(1) Prightarrow Qtag{Hypothesis}$$
                    $$(2) Qrightarrow Rtag{Hypothesis}$$
                    $$quadquadunderline{(3); Pquad }tag{Assumption}$$
                    $$quad quadquad |(4) Q tag{1 and 3: Modus Ponens}$$
                    $$quadquadquad |(5) R tag{2 and 4: Modus Ponens}$$
                    $$(6) P rightarrow R tag{3 - 5: if P, then R}$$



                    $$therefore ((Prightarrow Q)land (Qrightarrow R))rightarrow (Pto R)$$


                    (Note: step 6 is sometimes justified by "conditional introduction": together with what you are given or have established, if by assuming P, you can derive R, then you have shown $P rightarrow R$).





                    Note: when I first learned propositional logic, once it was established (proven), we referred to the following "syllogism":
                    $$Pto Q\
                    underline{Qto R}\
                    therefore Pto Rquad$$
                    by citing it as the "Hypothetical Syllogism", for justification in future proofs.






                    share|cite|improve this answer











                    $endgroup$
















                      12












                      12








                      12





                      $begingroup$

                      $$(1) Prightarrow Qtag{Hypothesis}$$
                      $$(2) Qrightarrow Rtag{Hypothesis}$$
                      $$quadquadunderline{(3); Pquad }tag{Assumption}$$
                      $$quad quadquad |(4) Q tag{1 and 3: Modus Ponens}$$
                      $$quadquadquad |(5) R tag{2 and 4: Modus Ponens}$$
                      $$(6) P rightarrow R tag{3 - 5: if P, then R}$$



                      $$therefore ((Prightarrow Q)land (Qrightarrow R))rightarrow (Pto R)$$


                      (Note: step 6 is sometimes justified by "conditional introduction": together with what you are given or have established, if by assuming P, you can derive R, then you have shown $P rightarrow R$).





                      Note: when I first learned propositional logic, once it was established (proven), we referred to the following "syllogism":
                      $$Pto Q\
                      underline{Qto R}\
                      therefore Pto Rquad$$
                      by citing it as the "Hypothetical Syllogism", for justification in future proofs.






                      share|cite|improve this answer











                      $endgroup$



                      $$(1) Prightarrow Qtag{Hypothesis}$$
                      $$(2) Qrightarrow Rtag{Hypothesis}$$
                      $$quadquadunderline{(3); Pquad }tag{Assumption}$$
                      $$quad quadquad |(4) Q tag{1 and 3: Modus Ponens}$$
                      $$quadquadquad |(5) R tag{2 and 4: Modus Ponens}$$
                      $$(6) P rightarrow R tag{3 - 5: if P, then R}$$



                      $$therefore ((Prightarrow Q)land (Qrightarrow R))rightarrow (Pto R)$$


                      (Note: step 6 is sometimes justified by "conditional introduction": together with what you are given or have established, if by assuming P, you can derive R, then you have shown $P rightarrow R$).





                      Note: when I first learned propositional logic, once it was established (proven), we referred to the following "syllogism":
                      $$Pto Q\
                      underline{Qto R}\
                      therefore Pto Rquad$$
                      by citing it as the "Hypothetical Syllogism", for justification in future proofs.







                      share|cite|improve this answer














                      share|cite|improve this answer



                      share|cite|improve this answer








                      edited Jan 17 '13 at 20:56

























                      answered Jan 17 '13 at 18:41









                      NamasteNamaste

                      1




                      1























                          4












                          $begingroup$

                          Since the logic you are using is not quantified, and there are only three variables, you can prove the argument valid with an eight row truth table covering all of the possible truth values of the three variables:



                          P  Q  R    Premise:  P -> Q    Premise:  Q -> R:    Conclusion:  P -> R
                          ------------------------------------------------------------------------
                          0 0 0 1 1 1
                          0 0 1 1 1 1
                          0 1 0 1 0 1
                          0 1 1 1 1 1
                          1 0 0 0 1 0
                          1 0 1 0 1 1
                          1 1 0 1 0 0
                          1 1 1 1 1 1


                          Now an argument is valid if the conclusion is true whenever all the premises are true.
                          This means that in all the rows where both premises have a 1, we check whether the conclusion has a 1. By golly, this is the case. Therefore the argument is valid!



                          If we found a zero, that would be a counterexample which destroys the argument: a situation where the premises are all true, yet the conclusion is false.



                          In my table, the P -> Q column and its siblings are simply derived from the truth table for the conditional. P -> Q is only false when P is true, and Q is false, and true for the other three possible values of the variables.






                          share|cite|improve this answer









                          $endgroup$


















                            4












                            $begingroup$

                            Since the logic you are using is not quantified, and there are only three variables, you can prove the argument valid with an eight row truth table covering all of the possible truth values of the three variables:



                            P  Q  R    Premise:  P -> Q    Premise:  Q -> R:    Conclusion:  P -> R
                            ------------------------------------------------------------------------
                            0 0 0 1 1 1
                            0 0 1 1 1 1
                            0 1 0 1 0 1
                            0 1 1 1 1 1
                            1 0 0 0 1 0
                            1 0 1 0 1 1
                            1 1 0 1 0 0
                            1 1 1 1 1 1


                            Now an argument is valid if the conclusion is true whenever all the premises are true.
                            This means that in all the rows where both premises have a 1, we check whether the conclusion has a 1. By golly, this is the case. Therefore the argument is valid!



                            If we found a zero, that would be a counterexample which destroys the argument: a situation where the premises are all true, yet the conclusion is false.



                            In my table, the P -> Q column and its siblings are simply derived from the truth table for the conditional. P -> Q is only false when P is true, and Q is false, and true for the other three possible values of the variables.






                            share|cite|improve this answer









                            $endgroup$
















                              4












                              4








                              4





                              $begingroup$

                              Since the logic you are using is not quantified, and there are only three variables, you can prove the argument valid with an eight row truth table covering all of the possible truth values of the three variables:



                              P  Q  R    Premise:  P -> Q    Premise:  Q -> R:    Conclusion:  P -> R
                              ------------------------------------------------------------------------
                              0 0 0 1 1 1
                              0 0 1 1 1 1
                              0 1 0 1 0 1
                              0 1 1 1 1 1
                              1 0 0 0 1 0
                              1 0 1 0 1 1
                              1 1 0 1 0 0
                              1 1 1 1 1 1


                              Now an argument is valid if the conclusion is true whenever all the premises are true.
                              This means that in all the rows where both premises have a 1, we check whether the conclusion has a 1. By golly, this is the case. Therefore the argument is valid!



                              If we found a zero, that would be a counterexample which destroys the argument: a situation where the premises are all true, yet the conclusion is false.



                              In my table, the P -> Q column and its siblings are simply derived from the truth table for the conditional. P -> Q is only false when P is true, and Q is false, and true for the other three possible values of the variables.






                              share|cite|improve this answer









                              $endgroup$



                              Since the logic you are using is not quantified, and there are only three variables, you can prove the argument valid with an eight row truth table covering all of the possible truth values of the three variables:



                              P  Q  R    Premise:  P -> Q    Premise:  Q -> R:    Conclusion:  P -> R
                              ------------------------------------------------------------------------
                              0 0 0 1 1 1
                              0 0 1 1 1 1
                              0 1 0 1 0 1
                              0 1 1 1 1 1
                              1 0 0 0 1 0
                              1 0 1 0 1 1
                              1 1 0 1 0 0
                              1 1 1 1 1 1


                              Now an argument is valid if the conclusion is true whenever all the premises are true.
                              This means that in all the rows where both premises have a 1, we check whether the conclusion has a 1. By golly, this is the case. Therefore the argument is valid!



                              If we found a zero, that would be a counterexample which destroys the argument: a situation where the premises are all true, yet the conclusion is false.



                              In my table, the P -> Q column and its siblings are simply derived from the truth table for the conditional. P -> Q is only false when P is true, and Q is false, and true for the other three possible values of the variables.







                              share|cite|improve this answer












                              share|cite|improve this answer



                              share|cite|improve this answer










                              answered Jan 18 '13 at 1:31









                              KazKaz

                              6,28311428




                              6,28311428























                                  3












                                  $begingroup$

                                  I'm not sure about what notations you might use nor if you're using a natural deductive system, but this is the idea behind what you asked:



                                  Suppose $Pto Q\
                                  Qto R\$ are true.



                                  We want to prove that $Pto R$ is true.



                                  To do this suppose $P$ is true.



                                  Because $Pto Q$ is true it follows that $Q$ is true.
                                  Now because $Q$ is true, from $Qto R$ being true follows that $R$ is true.



                                  We assumed $P$ was true and we deduced that $R$ is also true, therefore $Pto R$ as we wanted.






                                  share|cite|improve this answer











                                  $endgroup$













                                  • $begingroup$
                                    Is this just a form of proof by cases? I guess it's one half of it anyhow - knowing that if we assume $neg P$, the implication will be true no matter what - it's only relevant if we show that it's true if P is true?
                                    $endgroup$
                                    – agent154
                                    Jan 17 '13 at 18:28










                                  • $begingroup$
                                    No, it's not a proof by cases. What you mentioned is one way to do it: you always have $neg P vee P$, so you do it by cases the way you mentioned. However if you wanna prove a conditional statement, i.e., a statement of the form $Ato B$, it suffices to assume $A$ is true and then somehow deduce that $B$ is true.
                                    $endgroup$
                                    – Git Gud
                                    Jan 17 '13 at 18:32












                                  • $begingroup$
                                    I just realized - I could convert the implications to clausal form using the rule $(neg Pvee Q)wedge (neg Qvee R)Rightarrow (neg Pvee R)$ and then use the resolution rule to resolve out $(neg Qvee Q)$ to get $neg Pvee R$... That works as well.
                                    $endgroup$
                                    – agent154
                                    Jan 17 '13 at 18:38












                                  • $begingroup$
                                    Yes, that works too.
                                    $endgroup$
                                    – Git Gud
                                    Jan 17 '13 at 18:39
















                                  3












                                  $begingroup$

                                  I'm not sure about what notations you might use nor if you're using a natural deductive system, but this is the idea behind what you asked:



                                  Suppose $Pto Q\
                                  Qto R\$ are true.



                                  We want to prove that $Pto R$ is true.



                                  To do this suppose $P$ is true.



                                  Because $Pto Q$ is true it follows that $Q$ is true.
                                  Now because $Q$ is true, from $Qto R$ being true follows that $R$ is true.



                                  We assumed $P$ was true and we deduced that $R$ is also true, therefore $Pto R$ as we wanted.






                                  share|cite|improve this answer











                                  $endgroup$













                                  • $begingroup$
                                    Is this just a form of proof by cases? I guess it's one half of it anyhow - knowing that if we assume $neg P$, the implication will be true no matter what - it's only relevant if we show that it's true if P is true?
                                    $endgroup$
                                    – agent154
                                    Jan 17 '13 at 18:28










                                  • $begingroup$
                                    No, it's not a proof by cases. What you mentioned is one way to do it: you always have $neg P vee P$, so you do it by cases the way you mentioned. However if you wanna prove a conditional statement, i.e., a statement of the form $Ato B$, it suffices to assume $A$ is true and then somehow deduce that $B$ is true.
                                    $endgroup$
                                    – Git Gud
                                    Jan 17 '13 at 18:32












                                  • $begingroup$
                                    I just realized - I could convert the implications to clausal form using the rule $(neg Pvee Q)wedge (neg Qvee R)Rightarrow (neg Pvee R)$ and then use the resolution rule to resolve out $(neg Qvee Q)$ to get $neg Pvee R$... That works as well.
                                    $endgroup$
                                    – agent154
                                    Jan 17 '13 at 18:38












                                  • $begingroup$
                                    Yes, that works too.
                                    $endgroup$
                                    – Git Gud
                                    Jan 17 '13 at 18:39














                                  3












                                  3








                                  3





                                  $begingroup$

                                  I'm not sure about what notations you might use nor if you're using a natural deductive system, but this is the idea behind what you asked:



                                  Suppose $Pto Q\
                                  Qto R\$ are true.



                                  We want to prove that $Pto R$ is true.



                                  To do this suppose $P$ is true.



                                  Because $Pto Q$ is true it follows that $Q$ is true.
                                  Now because $Q$ is true, from $Qto R$ being true follows that $R$ is true.



                                  We assumed $P$ was true and we deduced that $R$ is also true, therefore $Pto R$ as we wanted.






                                  share|cite|improve this answer











                                  $endgroup$



                                  I'm not sure about what notations you might use nor if you're using a natural deductive system, but this is the idea behind what you asked:



                                  Suppose $Pto Q\
                                  Qto R\$ are true.



                                  We want to prove that $Pto R$ is true.



                                  To do this suppose $P$ is true.



                                  Because $Pto Q$ is true it follows that $Q$ is true.
                                  Now because $Q$ is true, from $Qto R$ being true follows that $R$ is true.



                                  We assumed $P$ was true and we deduced that $R$ is also true, therefore $Pto R$ as we wanted.







                                  share|cite|improve this answer














                                  share|cite|improve this answer



                                  share|cite|improve this answer








                                  edited Jan 18 '13 at 16:51

























                                  answered Jan 17 '13 at 18:23









                                  Git GudGit Gud

                                  28.9k1050101




                                  28.9k1050101












                                  • $begingroup$
                                    Is this just a form of proof by cases? I guess it's one half of it anyhow - knowing that if we assume $neg P$, the implication will be true no matter what - it's only relevant if we show that it's true if P is true?
                                    $endgroup$
                                    – agent154
                                    Jan 17 '13 at 18:28










                                  • $begingroup$
                                    No, it's not a proof by cases. What you mentioned is one way to do it: you always have $neg P vee P$, so you do it by cases the way you mentioned. However if you wanna prove a conditional statement, i.e., a statement of the form $Ato B$, it suffices to assume $A$ is true and then somehow deduce that $B$ is true.
                                    $endgroup$
                                    – Git Gud
                                    Jan 17 '13 at 18:32












                                  • $begingroup$
                                    I just realized - I could convert the implications to clausal form using the rule $(neg Pvee Q)wedge (neg Qvee R)Rightarrow (neg Pvee R)$ and then use the resolution rule to resolve out $(neg Qvee Q)$ to get $neg Pvee R$... That works as well.
                                    $endgroup$
                                    – agent154
                                    Jan 17 '13 at 18:38












                                  • $begingroup$
                                    Yes, that works too.
                                    $endgroup$
                                    – Git Gud
                                    Jan 17 '13 at 18:39


















                                  • $begingroup$
                                    Is this just a form of proof by cases? I guess it's one half of it anyhow - knowing that if we assume $neg P$, the implication will be true no matter what - it's only relevant if we show that it's true if P is true?
                                    $endgroup$
                                    – agent154
                                    Jan 17 '13 at 18:28










                                  • $begingroup$
                                    No, it's not a proof by cases. What you mentioned is one way to do it: you always have $neg P vee P$, so you do it by cases the way you mentioned. However if you wanna prove a conditional statement, i.e., a statement of the form $Ato B$, it suffices to assume $A$ is true and then somehow deduce that $B$ is true.
                                    $endgroup$
                                    – Git Gud
                                    Jan 17 '13 at 18:32












                                  • $begingroup$
                                    I just realized - I could convert the implications to clausal form using the rule $(neg Pvee Q)wedge (neg Qvee R)Rightarrow (neg Pvee R)$ and then use the resolution rule to resolve out $(neg Qvee Q)$ to get $neg Pvee R$... That works as well.
                                    $endgroup$
                                    – agent154
                                    Jan 17 '13 at 18:38












                                  • $begingroup$
                                    Yes, that works too.
                                    $endgroup$
                                    – Git Gud
                                    Jan 17 '13 at 18:39
















                                  $begingroup$
                                  Is this just a form of proof by cases? I guess it's one half of it anyhow - knowing that if we assume $neg P$, the implication will be true no matter what - it's only relevant if we show that it's true if P is true?
                                  $endgroup$
                                  – agent154
                                  Jan 17 '13 at 18:28




                                  $begingroup$
                                  Is this just a form of proof by cases? I guess it's one half of it anyhow - knowing that if we assume $neg P$, the implication will be true no matter what - it's only relevant if we show that it's true if P is true?
                                  $endgroup$
                                  – agent154
                                  Jan 17 '13 at 18:28












                                  $begingroup$
                                  No, it's not a proof by cases. What you mentioned is one way to do it: you always have $neg P vee P$, so you do it by cases the way you mentioned. However if you wanna prove a conditional statement, i.e., a statement of the form $Ato B$, it suffices to assume $A$ is true and then somehow deduce that $B$ is true.
                                  $endgroup$
                                  – Git Gud
                                  Jan 17 '13 at 18:32






                                  $begingroup$
                                  No, it's not a proof by cases. What you mentioned is one way to do it: you always have $neg P vee P$, so you do it by cases the way you mentioned. However if you wanna prove a conditional statement, i.e., a statement of the form $Ato B$, it suffices to assume $A$ is true and then somehow deduce that $B$ is true.
                                  $endgroup$
                                  – Git Gud
                                  Jan 17 '13 at 18:32














                                  $begingroup$
                                  I just realized - I could convert the implications to clausal form using the rule $(neg Pvee Q)wedge (neg Qvee R)Rightarrow (neg Pvee R)$ and then use the resolution rule to resolve out $(neg Qvee Q)$ to get $neg Pvee R$... That works as well.
                                  $endgroup$
                                  – agent154
                                  Jan 17 '13 at 18:38






                                  $begingroup$
                                  I just realized - I could convert the implications to clausal form using the rule $(neg Pvee Q)wedge (neg Qvee R)Rightarrow (neg Pvee R)$ and then use the resolution rule to resolve out $(neg Qvee Q)$ to get $neg Pvee R$... That works as well.
                                  $endgroup$
                                  – agent154
                                  Jan 17 '13 at 18:38














                                  $begingroup$
                                  Yes, that works too.
                                  $endgroup$
                                  – Git Gud
                                  Jan 17 '13 at 18:39




                                  $begingroup$
                                  Yes, that works too.
                                  $endgroup$
                                  – Git Gud
                                  Jan 17 '13 at 18:39











                                  1












                                  $begingroup$

                                  Although it is late, I wanted to give a simple and convincing demonstration:



                                  (1) P -> Q (Hypothesis)
                                  (2) Q -> R (Hypothesis)
                                  (3) ¬Q -> ¬P (Contraposition (1))
                                  (4) (Q -> R) ∧ (¬Q -> ¬P) (Union (2) and (3))
                                  (5) Q ∨ ¬Q (Law of excluded middle)
                                  (6) (Q -> R) ∧ (¬Q -> ¬P) ∧ ( Q ∨ ¬Q) (Union (4) and (5) )
                                  ____________________________
                                  (R ∨ ¬P) (Dilemma of (6), P -> R=def.(R ∨ ¬P) Definition of logical implication)





                                  share|cite|improve this answer









                                  $endgroup$


















                                    1












                                    $begingroup$

                                    Although it is late, I wanted to give a simple and convincing demonstration:



                                    (1) P -> Q (Hypothesis)
                                    (2) Q -> R (Hypothesis)
                                    (3) ¬Q -> ¬P (Contraposition (1))
                                    (4) (Q -> R) ∧ (¬Q -> ¬P) (Union (2) and (3))
                                    (5) Q ∨ ¬Q (Law of excluded middle)
                                    (6) (Q -> R) ∧ (¬Q -> ¬P) ∧ ( Q ∨ ¬Q) (Union (4) and (5) )
                                    ____________________________
                                    (R ∨ ¬P) (Dilemma of (6), P -> R=def.(R ∨ ¬P) Definition of logical implication)





                                    share|cite|improve this answer









                                    $endgroup$
















                                      1












                                      1








                                      1





                                      $begingroup$

                                      Although it is late, I wanted to give a simple and convincing demonstration:



                                      (1) P -> Q (Hypothesis)
                                      (2) Q -> R (Hypothesis)
                                      (3) ¬Q -> ¬P (Contraposition (1))
                                      (4) (Q -> R) ∧ (¬Q -> ¬P) (Union (2) and (3))
                                      (5) Q ∨ ¬Q (Law of excluded middle)
                                      (6) (Q -> R) ∧ (¬Q -> ¬P) ∧ ( Q ∨ ¬Q) (Union (4) and (5) )
                                      ____________________________
                                      (R ∨ ¬P) (Dilemma of (6), P -> R=def.(R ∨ ¬P) Definition of logical implication)





                                      share|cite|improve this answer









                                      $endgroup$



                                      Although it is late, I wanted to give a simple and convincing demonstration:



                                      (1) P -> Q (Hypothesis)
                                      (2) Q -> R (Hypothesis)
                                      (3) ¬Q -> ¬P (Contraposition (1))
                                      (4) (Q -> R) ∧ (¬Q -> ¬P) (Union (2) and (3))
                                      (5) Q ∨ ¬Q (Law of excluded middle)
                                      (6) (Q -> R) ∧ (¬Q -> ¬P) ∧ ( Q ∨ ¬Q) (Union (4) and (5) )
                                      ____________________________
                                      (R ∨ ¬P) (Dilemma of (6), P -> R=def.(R ∨ ¬P) Definition of logical implication)






                                      share|cite|improve this answer












                                      share|cite|improve this answer



                                      share|cite|improve this answer










                                      answered Jun 2 '17 at 5:49









                                      Roger Figueroa QuinteroRoger Figueroa Quintero

                                      749




                                      749























                                          1












                                          $begingroup$

                                          Denote by $+$ the $or$ logic operator and by $cdot$ the $and$ logic operator.



                                          We have $(ARightarrow B)=bar A+B$ and $(BRightarrow C) =bar B+C$



                                          Assume $(bar A + B)=1=(bar B+C)$.



                                          Then
                                          $1=1cdot 1=(bar A + B)cdot (bar B+C) \
                                          = (bar Abar B + bar A C + Bbar B + BC)\
                                          = bar A + BC
                                          = (bar A + B) cdot (bar A+C)
                                          \
                                          =bar A + C
                                          $






                                          share|cite|improve this answer











                                          $endgroup$













                                          • $begingroup$
                                            Ha! This is a great argument
                                            $endgroup$
                                            – Ben Kushigian
                                            Dec 23 '18 at 17:17
















                                          1












                                          $begingroup$

                                          Denote by $+$ the $or$ logic operator and by $cdot$ the $and$ logic operator.



                                          We have $(ARightarrow B)=bar A+B$ and $(BRightarrow C) =bar B+C$



                                          Assume $(bar A + B)=1=(bar B+C)$.



                                          Then
                                          $1=1cdot 1=(bar A + B)cdot (bar B+C) \
                                          = (bar Abar B + bar A C + Bbar B + BC)\
                                          = bar A + BC
                                          = (bar A + B) cdot (bar A+C)
                                          \
                                          =bar A + C
                                          $






                                          share|cite|improve this answer











                                          $endgroup$













                                          • $begingroup$
                                            Ha! This is a great argument
                                            $endgroup$
                                            – Ben Kushigian
                                            Dec 23 '18 at 17:17














                                          1












                                          1








                                          1





                                          $begingroup$

                                          Denote by $+$ the $or$ logic operator and by $cdot$ the $and$ logic operator.



                                          We have $(ARightarrow B)=bar A+B$ and $(BRightarrow C) =bar B+C$



                                          Assume $(bar A + B)=1=(bar B+C)$.



                                          Then
                                          $1=1cdot 1=(bar A + B)cdot (bar B+C) \
                                          = (bar Abar B + bar A C + Bbar B + BC)\
                                          = bar A + BC
                                          = (bar A + B) cdot (bar A+C)
                                          \
                                          =bar A + C
                                          $






                                          share|cite|improve this answer











                                          $endgroup$



                                          Denote by $+$ the $or$ logic operator and by $cdot$ the $and$ logic operator.



                                          We have $(ARightarrow B)=bar A+B$ and $(BRightarrow C) =bar B+C$



                                          Assume $(bar A + B)=1=(bar B+C)$.



                                          Then
                                          $1=1cdot 1=(bar A + B)cdot (bar B+C) \
                                          = (bar Abar B + bar A C + Bbar B + BC)\
                                          = bar A + BC
                                          = (bar A + B) cdot (bar A+C)
                                          \
                                          =bar A + C
                                          $







                                          share|cite|improve this answer














                                          share|cite|improve this answer



                                          share|cite|improve this answer








                                          edited Dec 23 '18 at 17:40









                                          Ben Kushigian

                                          1879




                                          1879










                                          answered Nov 7 '17 at 14:17









                                          SmiliaSmilia

                                          733617




                                          733617












                                          • $begingroup$
                                            Ha! This is a great argument
                                            $endgroup$
                                            – Ben Kushigian
                                            Dec 23 '18 at 17:17


















                                          • $begingroup$
                                            Ha! This is a great argument
                                            $endgroup$
                                            – Ben Kushigian
                                            Dec 23 '18 at 17:17
















                                          $begingroup$
                                          Ha! This is a great argument
                                          $endgroup$
                                          – Ben Kushigian
                                          Dec 23 '18 at 17:17




                                          $begingroup$
                                          Ha! This is a great argument
                                          $endgroup$
                                          – Ben Kushigian
                                          Dec 23 '18 at 17:17











                                          0












                                          $begingroup$

                                          The answer for $6$ is indeed, true but if you had to show it with a truth table would be as follows



                                          $$begin{array}{cccccccc}P & Q & R & P!rightarrow! Q & Q!rightarrow! R & P!rightarrow! R & (P!rightarrow! Q)!wedge!(Q!rightarrow! R) & [(P!rightarrow! Q)!wedge!(Q!rightarrow! R)]!rightarrow! (P!rightarrow! R)\
                                          T & T & T & T & T & T & T & T\
                                          T & T & F & T & F & F & F & T\
                                          T & F & T & F & T & T & F & T\
                                          T & F & F & F & T & F & F & T\
                                          F & T & T & T & T & T & T & T\
                                          F & T & F & T & F & T & F & T\
                                          F & F & T & T & T & T & T & T\
                                          F & F & F & T & T & T & T & Tend{array}$$





                                          As you notice above the column $(P!rightarrow! Q)!wedge!(Q!rightarrow! R)$ is the same as the column of $(P!rightarrow! R).$ So with both the hypotheses $(P!rightarrow! R)$ and $(Q!rightarrow! R)$ we can conclude that $(P!rightarrow! R).$






                                          share|cite|improve this answer











                                          $endgroup$


















                                            0












                                            $begingroup$

                                            The answer for $6$ is indeed, true but if you had to show it with a truth table would be as follows



                                            $$begin{array}{cccccccc}P & Q & R & P!rightarrow! Q & Q!rightarrow! R & P!rightarrow! R & (P!rightarrow! Q)!wedge!(Q!rightarrow! R) & [(P!rightarrow! Q)!wedge!(Q!rightarrow! R)]!rightarrow! (P!rightarrow! R)\
                                            T & T & T & T & T & T & T & T\
                                            T & T & F & T & F & F & F & T\
                                            T & F & T & F & T & T & F & T\
                                            T & F & F & F & T & F & F & T\
                                            F & T & T & T & T & T & T & T\
                                            F & T & F & T & F & T & F & T\
                                            F & F & T & T & T & T & T & T\
                                            F & F & F & T & T & T & T & Tend{array}$$





                                            As you notice above the column $(P!rightarrow! Q)!wedge!(Q!rightarrow! R)$ is the same as the column of $(P!rightarrow! R).$ So with both the hypotheses $(P!rightarrow! R)$ and $(Q!rightarrow! R)$ we can conclude that $(P!rightarrow! R).$






                                            share|cite|improve this answer











                                            $endgroup$
















                                              0












                                              0








                                              0





                                              $begingroup$

                                              The answer for $6$ is indeed, true but if you had to show it with a truth table would be as follows



                                              $$begin{array}{cccccccc}P & Q & R & P!rightarrow! Q & Q!rightarrow! R & P!rightarrow! R & (P!rightarrow! Q)!wedge!(Q!rightarrow! R) & [(P!rightarrow! Q)!wedge!(Q!rightarrow! R)]!rightarrow! (P!rightarrow! R)\
                                              T & T & T & T & T & T & T & T\
                                              T & T & F & T & F & F & F & T\
                                              T & F & T & F & T & T & F & T\
                                              T & F & F & F & T & F & F & T\
                                              F & T & T & T & T & T & T & T\
                                              F & T & F & T & F & T & F & T\
                                              F & F & T & T & T & T & T & T\
                                              F & F & F & T & T & T & T & Tend{array}$$





                                              As you notice above the column $(P!rightarrow! Q)!wedge!(Q!rightarrow! R)$ is the same as the column of $(P!rightarrow! R).$ So with both the hypotheses $(P!rightarrow! R)$ and $(Q!rightarrow! R)$ we can conclude that $(P!rightarrow! R).$






                                              share|cite|improve this answer











                                              $endgroup$



                                              The answer for $6$ is indeed, true but if you had to show it with a truth table would be as follows



                                              $$begin{array}{cccccccc}P & Q & R & P!rightarrow! Q & Q!rightarrow! R & P!rightarrow! R & (P!rightarrow! Q)!wedge!(Q!rightarrow! R) & [(P!rightarrow! Q)!wedge!(Q!rightarrow! R)]!rightarrow! (P!rightarrow! R)\
                                              T & T & T & T & T & T & T & T\
                                              T & T & F & T & F & F & F & T\
                                              T & F & T & F & T & T & F & T\
                                              T & F & F & F & T & F & F & T\
                                              F & T & T & T & T & T & T & T\
                                              F & T & F & T & F & T & F & T\
                                              F & F & T & T & T & T & T & T\
                                              F & F & F & T & T & T & T & Tend{array}$$





                                              As you notice above the column $(P!rightarrow! Q)!wedge!(Q!rightarrow! R)$ is the same as the column of $(P!rightarrow! R).$ So with both the hypotheses $(P!rightarrow! R)$ and $(Q!rightarrow! R)$ we can conclude that $(P!rightarrow! R).$







                                              share|cite|improve this answer














                                              share|cite|improve this answer



                                              share|cite|improve this answer








                                              edited Oct 27 '13 at 17:16









                                              Cameron Buie

                                              87k773161




                                              87k773161










                                              answered Oct 27 '13 at 16:43









                                              John SgutanJohn Sgutan

                                              11




                                              11






























                                                  draft saved

                                                  draft discarded




















































                                                  Thanks for contributing an answer to Mathematics Stack Exchange!


                                                  • Please be sure to answer the question. Provide details and share your research!

                                                  But avoid



                                                  • Asking for help, clarification, or responding to other answers.

                                                  • Making statements based on opinion; back them up with references or personal experience.


                                                  Use MathJax to format equations. MathJax reference.


                                                  To learn more, see our tips on writing great answers.




                                                  draft saved


                                                  draft discarded














                                                  StackExchange.ready(
                                                  function () {
                                                  StackExchange.openid.initPostLogin('.new-post-login', 'https%3a%2f%2fmath.stackexchange.com%2fquestions%2f280893%2fhow-do-i-prove-the-transitivity-of-a-set-of-implications%23new-answer', 'question_page');
                                                  }
                                                  );

                                                  Post as a guest















                                                  Required, but never shown





















































                                                  Required, but never shown














                                                  Required, but never shown












                                                  Required, but never shown







                                                  Required, but never shown

































                                                  Required, but never shown














                                                  Required, but never shown












                                                  Required, but never shown







                                                  Required, but never shown







                                                  Popular posts from this blog

                                                  Plaza Victoria

                                                  In PowerPoint, is there a keyboard shortcut for bulleted / numbered list?

                                                  How to put 3 figures in Latex with 2 figures side by side and 1 below these side by side images but in...